How Does the Chain Rule Apply in Polar Coordinates?

  • Thread starter Thread starter JohnTheMan
  • Start date Start date
  • Tags Tags
    Chain Chain rule
Click For Summary
The discussion focuses on applying the chain rule in polar coordinates to derive a specific equation involving second partial derivatives. The user attempts to compute the second derivative of z with respect to θ but finds discrepancies in their results compared to the problem statement. Key corrections highlight the need to include second derivatives of x and y with respect to θ in the calculations. The conversation emphasizes the importance of correctly applying the chain rule and ensuring all necessary derivatives are accounted for. Ultimately, the user acknowledges the clarification and expresses gratitude for the assistance.
JohnTheMan
Messages
3
Reaction score
0

Homework Statement


z = ƒ(x,y), x = rcos(θ), y = rsin(θ)
Use the chain rule to show that:
\frac{1}{r^{2}}\frac{\partial ^{2} z}{\partial \theta ^{2}} = sin^{2}(\theta)\frac{\partial ^{2} z}{\partial x^{2}}-2sin(\theta)cos(\theta)\frac{\partial ^{2} z}{\partial x \partial y}+cos^{2}(\theta)\frac{\partial ^{2} z}{\partial y ^{2}} - \frac{1}{r}\frac{\partial z}{\partial r}

Homework Equations


∂z/∂r = (∂z/∂x)(∂x/∂r) + (∂z/∂y)(∂y/∂r)

The Attempt at a Solution


I have worked it through and I have found that:
\frac{\partial z}{\partial \theta } = -rsin(\theta)\frac{\partial z}{\partial x}+rcos(\theta)\frac{\partial z}{\partial y}
\frac{\partial z}{\partial r } = cos(\theta)\frac{\partial z}{\partial x}+sin(\theta)\frac{\partial z}{\partial y}
\frac{\partial ^{2} z}{\partial \theta ^{2}} = r^{2}(sin^{2}(\theta)\frac{\partial ^{2} z}{\partial x^{2}}-2sin(\theta)cos(\theta)\frac{\partial ^{2} z}{\partial x \partial y}+cos^{2}(\theta)\frac{\partial ^{2} z}{\partial y ^{2}})
I have checked these a number of times and they seem right to me. The last one listed is the second partial derivative of z in terms of θ, but it doesn't match that of the problem statement. I'm not really sure what I'm doing wrong. Thanks :)
 
Physics news on Phys.org
Your second derivative looks like you just squared your first derivative.
You need to actually take one more derivative to get ## \frac{\partial^2 z}{\partial \theta^2}##.
 
RUber said:
Your second derivative looks like you just squared your first derivative.
You need to actually take one more derivative to get ## \frac{\partial^2 z}{\partial \theta^2}##.
I didn't square it, I took the second derivative.
 
You have the first part right. But your second derivative is wrong.
##\frac{\partial}{\partial \theta } f(x,y) = \frac{\partial f}{\partial x} \frac{\partial x}{\partial \theta} + \frac{\partial f}{\partial y} \frac{\partial y}{\partial \theta}##
##\frac{\partial}{\partial \theta }\left( \frac{\partial}{\partial \theta } f(x,y)\right) = \frac{\partial}{\partial \theta }\left( \frac{\partial f}{\partial x} \frac{\partial x}{\partial \theta} + \frac{\partial f}{\partial y} \frac{\partial y}{\partial \theta}\right) ##
Which is
## = \left( \frac{\partial}{\partial \theta }\frac{\partial f}{\partial x}\right) \frac{\partial x}{\partial \theta}+\underline{ \frac{\partial f}{\partial x} \left( \frac{\partial}{\partial \theta }\frac{\partial x}{\partial \theta} \right) } + \left( \frac{\partial}{\partial \theta } \frac{\partial f}{\partial y}\right) \frac{\partial y}{\partial \theta}+ \underline{ \frac{\partial f}{\partial y} \left( \frac{\partial}{\partial \theta }\frac{\partial y}{\partial \theta}\right)} ##
In your expansion, you missed the second derivatives with respect to theta on x and y. Which will give you the equivalent parts you need for the partial with respect to r.
 
  • Like
Likes JohnTheMan
RUber said:
You have the first part right. But your second derivative is wrong.
##\frac{\partial}{\partial \theta } f(x,y) = \frac{\partial f}{\partial x} \frac{\partial x}{\partial \theta} + \frac{\partial f}{\partial y} \frac{\partial y}{\partial \theta}##
##\frac{\partial}{\partial \theta }\left( \frac{\partial}{\partial \theta } f(x,y)\right) = \frac{\partial}{\partial \theta }\left( \frac{\partial f}{\partial x} \frac{\partial x}{\partial \theta} + \frac{\partial f}{\partial y} \frac{\partial y}{\partial \theta}\right) ##
Which is
## = \left( \frac{\partial}{\partial \theta }\frac{\partial f}{\partial x}\right) \frac{\partial x}{\partial \theta}+\underline{ \frac{\partial f}{\partial x} \left( \frac{\partial}{\partial \theta }\frac{\partial x}{\partial \theta} \right) } + \left( \frac{\partial}{\partial \theta } \frac{\partial f}{\partial y}\right) \frac{\partial y}{\partial \theta}+ \underline{ \frac{\partial f}{\partial y} \left( \frac{\partial}{\partial \theta }\frac{\partial y}{\partial \theta}\right)} ##
In your expansion, you missed the second derivatives with respect to theta on x and y. Which will give you the equivalent parts you need for the partial with respect to r.
RUber said:
You have the first part right. But your second derivative is wrong.
##\frac{\partial}{\partial \theta } f(x,y) = \frac{\partial f}{\partial x} \frac{\partial x}{\partial \theta} + \frac{\partial f}{\partial y} \frac{\partial y}{\partial \theta}##
##\frac{\partial}{\partial \theta }\left( \frac{\partial}{\partial \theta } f(x,y)\right) = \frac{\partial}{\partial \theta }\left( \frac{\partial f}{\partial x} \frac{\partial x}{\partial \theta} + \frac{\partial f}{\partial y} \frac{\partial y}{\partial \theta}\right) ##
Which is
## = \left( \frac{\partial}{\partial \theta }\frac{\partial f}{\partial x}\right) \frac{\partial x}{\partial \theta}+\underline{ \frac{\partial f}{\partial x} \left( \frac{\partial}{\partial \theta }\frac{\partial x}{\partial \theta} \right) } + \left( \frac{\partial}{\partial \theta } \frac{\partial f}{\partial y}\right) \frac{\partial y}{\partial \theta}+ \underline{ \frac{\partial f}{\partial y} \left( \frac{\partial}{\partial \theta }\frac{\partial y}{\partial \theta}\right)} ##
In your expansion, you missed the second derivatives with respect to theta on x and y. Which will give you the equivalent parts you need for the partial with respect to r.
Ahh. That makes sense. Thanks a lot.
 
Question: A clock's minute hand has length 4 and its hour hand has length 3. What is the distance between the tips at the moment when it is increasing most rapidly?(Putnam Exam Question) Answer: Making assumption that both the hands moves at constant angular velocities, the answer is ## \sqrt{7} .## But don't you think this assumption is somewhat doubtful and wrong?

Similar threads

  • · Replies 3 ·
Replies
3
Views
3K
  • · Replies 8 ·
Replies
8
Views
2K
  • · Replies 3 ·
Replies
3
Views
2K
  • · Replies 1 ·
Replies
1
Views
1K
  • · Replies 4 ·
Replies
4
Views
2K
  • · Replies 2 ·
Replies
2
Views
1K
Replies
3
Views
2K
Replies
7
Views
2K
  • · Replies 11 ·
Replies
11
Views
2K
  • · Replies 9 ·
Replies
9
Views
2K